Prove that a set with two properties has to be the positive real set

Click For Summary
The discussion revolves around proving that a set S with specific properties must be the set of positive real numbers. The properties state that if two elements are in S, their product and sum are also in S, and for every real number, it either belongs to S, is zero, or its negative is in S. The user attempts to show that S is a subset of the positive reals and vice versa, using theorems about positive numbers and their properties. They successfully demonstrate that if 1 is in S, then all positive integers must also be in S, but struggle to derive a contradiction for negative elements in S. The conversation highlights the challenge of proving the completeness of S as the set of positive numbers.
Perrault
Messages
13
Reaction score
0

Homework Statement



Let S ⊆ R be such that
(i) a, b ∈ S ⇒ ab, a + b ∈ S
(ii) for all x ∈ R exactly one of the following holds
x ∈ S, x = 0, −x ∈ S.

Show that S = {x ∈ R ; x > 0} (the set of positive numbers P)
2. Relevant theorems

(T1) a² > 0 ∀ a ∈ R. (So a²∈P)

(T2) All positive real numbers have a square root.

The Attempt at a Solution



This has been tantalizing me for hours. I want to prove that S⊆P and that P⊆S so that S=P. Trying to prove that S⊆P, I assume a∈S. Then a²∈S, by property (i), and a²∈P (by theorem T1). If we assume that a∈S is negative then we should be able to arrive at a contradiction, but I can't get to it! I understand that a and a² would be of different sign, but that doesn't seem to help me much.

I did show that if 1 is in S then it is in P and vice-versa. From there I am able to obtain the same thing for all positive integers (by property (i), if 1∈S then 1+1∈S so 2+1∈S, ...)

Anyone got any clue?

Thank you so much!
 
Physics news on Phys.org
If a < 0, consider sqrt(-a).
 
Question: A clock's minute hand has length 4 and its hour hand has length 3. What is the distance between the tips at the moment when it is increasing most rapidly?(Putnam Exam Question) Answer: Making assumption that both the hands moves at constant angular velocities, the answer is ## \sqrt{7} .## But don't you think this assumption is somewhat doubtful and wrong?

Similar threads

  • · Replies 14 ·
Replies
14
Views
2K
  • · Replies 1 ·
Replies
1
Views
1K
  • · Replies 3 ·
Replies
3
Views
2K
Replies
3
Views
2K
  • · Replies 1 ·
Replies
1
Views
2K
Replies
1
Views
2K
  • · Replies 9 ·
Replies
9
Views
2K
Replies
1
Views
1K
Replies
9
Views
2K
Replies
2
Views
1K